刘新悦 发表于 2023-5-3 20:53:23

求助万分感谢:LNG接收站储罐TGE罐容计算

求助大佬,求助
设计LNG接收站储罐部分,计算储罐容量,我选择的是TGE罐容计算方法,计算过程中有一个季节调峰量,题目中给了月不均匀系数表和管网分输用气量表。求助各位大佬,这个调峰量要怎么计算啊data:image/png;base64,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:image/png;base64,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

储罐设置需考虑LNG运输船船容、最大不可作业天数、季节调峰,如果大佬们有其他计算方法也可以。
万分感谢~


自由飞翔 发表于 2023-5-7 17:30:35

ChatGPT回答:

LNG接收站的储罐部分,需要考虑到LNG运输船船容、最大不可作业天数以及季节调峰。其中,季节调峰量的计算方法如下:

根据管网分输用气量表,计算出不同月份的平均用气量,单位为万立方米/天。

根据月不均匀系数表,计算出不同月份的月不均匀系数。

将平均用气量乘以月不均匀系数,得到每个月的实际用气量,单位为万立方米/天。

根据LNG的密度和热值,将每个月的用气量转换为对应的LNG储罐容量,单位为万立方米。

对于季节调峰量,可以取最高月用气量与最低月用气量之差,即为季节调峰量。

在计算储罐容量时,可以使用TGE罐容计算方法。该方法需要考虑到LNG运输船船容、最大不可作业天数、季节调峰等因素。具体的计算步骤如下:

计算LNG运输船船容。

确定最大不可作业天数,通常为30天。

计算每日最大进口量,包括日常用气量和季节调峰量。

根据LNG的密度和热值,将每日最大进口量转换为对应的LNG储罐容量,单位为万立方米。

将最大不可作业天数乘以每日最大进口量,得到LNG储罐的最小容量,单位为万立方米。

为了确保LNG储罐的安全运营,可以将最小容量乘以一个安全系数,通常为1.2至1.5之间,得到LNG储罐的实际容量,单位为万立方米。

需要注意的是,在进行LNG接收站储罐的设计时,还需要考虑到LNG的蒸发量、储罐的保温等因素,以确保LNG的安全运输和储存。


需要ChatGPT的联系我!

robinwqt 发表于 2023-7-27 14:00:20

需要仿真的联系我
页: [1]
查看完整版本: 求助万分感谢:LNG接收站储罐TGE罐容计算